Đến nội dung

Hình ảnh

BĐT AM-GM


  • Please log in to reply
Chủ đề này có 339 trả lời

#21
minhtuyb

minhtuyb

    Giả ngu chuyên nghiệp

  • Thành viên
  • 470 Bài viết

Nhìn phát thấy ngay là $Holder$ Hình đã gửi
---
-Nếu $\exists a_i+b_i=0\Rightarrow a_i=b_i=0\Rightarrow VT(*)=VP(*)$
-Nếu các biến đều dương: áp dụng BĐT $AM-GM$ cho $n$ số, ta có:
$$\dfrac{a_1}{a_1+b_1}+\dfrac{a_2}{a_2+b_2}+...+\dfrac{a_n}{a_n+b_n}\ge \dfrac{n\sqrt[n]{a_1a_2...a_n}}{(a_{1}+b_{1})(a_{2}+b_{2})...(a_{n}+b_{n})}\\
\dfrac{b_1}{a_1+b_1}+\dfrac{b_2}{a_2+b_2}+...+\dfrac{b_n}{a_n+b_n}\ge \dfrac{n\sqrt[n]{b_1b_2...b_n}}{(a_{1}+b_{1})(a_{2}+b_{2})...(a_{n}+b_{n})}$$
Cộng vế với vế của 2 BĐT cùng chiều trên, sau vài bước biến đổi ta có ĐPCM. Dấu bằng xảy ra chẳng hạn khi $a_1=a_2=...=a_n;b_1=b_2=...=b_n\ \square$
---
Hai bài trên là hệ quả trực tiếp của BĐT này

mọi người ta tập trung lại 1 tí nhé,mình post bài có kèm theo bài tập là muốn mọi người vận dụng thẳng các kiến thức trên vào bài tập cho vững luôn,dĩ nhiên mình không nề nà gì việc đăng thêm bài mới,nhưng mình nghĩ,việc đó nên để khi ta đã giải quyết trọn vẹn các bài tập
thân
tình hình là ta còn nhiều bài tồn đọng quá
ĐỀ NGHỊ
bài 4 phần 1(kiến thức đã nêu,mong mọi người triệt để áp dụng :wacko:
với mọi a.b không âm,chứng minh
$\sqrt[3]{\frac{a}{b}}+\sqrt[3]{\frac{b}{a}}\leq \sqrt[3]{2(a+b)(\frac{1}{a}+\frac{1}{b})}$

Ô đề nghị này ko ai làm á :ohmy:
---
Áp dụng cái mình c/m đằng trước với $n=3$, ta có:
$$RHS=\sqrt[3]{(1+1)(a+b)(\dfrac{1}{b}+\dfrac{1}{a})}\ge \sqrt[3]{1.a.\dfrac{1}{b}}+\sqrt[3]{1.b.\dfrac{1}{a} }\\ \Rightarrow Q.E.D$$
---
Mn ủng hộ topic đi chứ nhỉ ~O)

Bài viết đã được chỉnh sửa nội dung bởi minhtuyb: 24-11-2012 - 23:06

Phấn đấu vì tương lai con em chúng ta!

#22
19kvh97

19kvh97

    Sĩ quan

  • Thành viên
  • 423 Bài viết
Mình xin đóng góp cho topic bài này
Bài 5: Cho $a,b,c>0$ và $a+b+c=3$. CMR
$\sum \frac{x}{x^4+y+z}\leq 1$

Bài viết đã được chỉnh sửa nội dung bởi 19kvh97: 25-11-2012 - 11:10


#23
minhtuyb

minhtuyb

    Giả ngu chuyên nghiệp

  • Thành viên
  • 470 Bài viết

c,Đặt ẩn phụ
Vì kĩ thuật đặt ăn phụ mình đã trình bày nên xin chỉ nêu 1 VD nho nhỏ sau
Chứng mnih với mọi a,b,c dương,ta có
$\frac{1}{a\sqrt{a+b}}+\frac{1}{b\sqrt{b+c}}+\frac{1}{c\sqrt{c+a}}\geq \frac{3}{\sqrt{2abc}}$
Giải BĐT cần chứng minh tương đương $\sqrt{\frac{2bc}{a(a+b)}}+\sqrt{\frac{2ca}{b(b+c)}}+\sqrt{\frac{2ab}{c(c+a)}}\geq 3$
Ta sẽ đặt $\sqrt{\frac{2bc}{a(a+b)}}=x;\sqrt{\frac{2ca}{b(b+c)}}=y;\sqrt{\frac{2ab}{c(c+a)}}=z$
Chú ý thêm rằng xy=$\frac{2c}{\sqrt{(a+b)(b+c)}}$,ta sẽ chứng minh $xy+yz+zx\geq 3$
Tiếp tục đặt $\sqrt{a+b}=p,\sqrt{b+c}=q,\sqrt{c+a}=r$,khi đó,dề thấy $xy=\frac{p^2+q^2-r^2}{qr}$
BĐT cần chứng minh sẽ là $(p^3+q^3+r^3)(p^2q+q^2r+r^2p)\geq q^2p+r^2q+p^2r+3pqr$(Đây là 1 BĐT khá hay,xin nhường cho các bạn)

Hix $p,q,r$ trâu bò quá ="=. Mình xin đóng góp lời giải theo mình nghĩ là hay hơn :lol: :
---
BĐT cần chứng minh tương đương:
$$\sum \sqrt{\frac{2bc}{a(a+b)}}\geq 3\ (*)$$
Ta có:
$$LHS(*)=\sum \dfrac{2bc}{\sqrt{2abc(a+b)}}=\sum \dfrac{2bc}{\sqrt{2ab(ac+bc)}}\ge \sum \dfrac{4bc}{2ab+ac+bc}$$
Đặt $x=ab;y=bc;z=ca\Rightarrow x,y,z>0$. Ta cần c/m:
$$\sum \dfrac{4x}{x+2y+z}\ge 3\Leftrightarrow \sum \dfrac{x}{x+2y+z}\ge \dfrac{3}{4}\ (**)$$

(Đến đây nếu dùng $Schwarz$ thì ra luôn. Nhưng vì là topic $AM-GM$ nên ta xài luôn $AM-GM$ :icon6: )

Chuẩn hoá: $x+y+z=3$. Viết lại $(**)$:
$$\sum \dfrac{x}{y+3}\ge \dfrac{3}{4}$$
Đúng từ các BĐT sau:
$$\dfrac{x}{y+3}+\dfrac{x(y+3)}{16}\ge \dfrac{x}{2}\\ \dfrac{y}{z+3}+\dfrac{y(z+3)}{16}\ge \dfrac{y}{2}\\\dfrac{z}{x+3}+\dfrac{z(x+3)}{16}\ge \dfrac{z}{2}\\ \dfrac{(x+y+z)^2}{3.16}\ge \dfrac{xy+yz+zx}{16}$$
Vậy $(**)$ đúng $\Rightarrow (*)$ đúng.
Dấu bằng xảy ra khi $a=b=c$
Phép c/m hoàn tất $\square$.
Phấn đấu vì tương lai con em chúng ta!

#24
minhtuyb

minhtuyb

    Giả ngu chuyên nghiệp

  • Thành viên
  • 470 Bài viết

Mình xin đóng góp cho topic bài này
Bài 5: Cho $a,b,c>0$ và $a+b+c=3$. CMR
$\sum \frac{x}{x^4+y+z}\geq 1$

Mình lại c/m được $\sum \frac{x}{x^4+y+z}\leq 1\ (*)$ Hình đã gửi
---
Vời giả thiết trên thì: $\dfrac{x}{x^4+y+z}=\dfrac{x}{x^4+3-x}$
Ta sẽ c/m: $\dfrac{x}{x^4+3-x}\le \dfrac{1}{3}\Leftrightarrow x^4-4x+3\ge 0\Leftrightarrow (x-1)^2(x^2+2x+3)\ge 0$ (Luôn đúng)
C/m tt rồi cộng lại ta suy ra $(*)$ đúng.
Dấu bằng xảy ra khi $x=y=z=1\ \square$
---
Thôi đi ngủ đây, lần sau bạn nhớ post bài nào liên quan tới topic nhé Hình đã gửi

Bài viết đã được chỉnh sửa nội dung bởi minhtuyb: 25-11-2012 - 00:17

Phấn đấu vì tương lai con em chúng ta!

#25
19kvh97

19kvh97

    Sĩ quan

  • Thành viên
  • 423 Bài viết

Mình lại c/m được $\sum \frac{x}{x^4+y+z}\leq 1\ (*)$ Hình đã gửi
---
Vời giả thiết trên thì: $\dfrac{x}{x^4+y+z}=\dfrac{x}{x^4+3-x}$
Ta sẽ c/m: $\dfrac{x}{x^4+3-x}\le \dfrac{1}{3}\Leftrightarrow x^4-4x+3\ge 0\Leftrightarrow (x-1)^2(x^2+2x+3)\ge 0$ (Luôn đúng)
C/m tt rồi cộng lại ta suy ra $(*)$ đúng.
Dấu bằng xảy ra khi $x=y=z=1\ \square$
---
Thôi đi ngủ đây, lần sau bạn nhớ post bài nào liên quan tới topic nhé Hình đã gửi

xin lỗi mình nhầm dấu mình đã sửa lại rồi đó
C2 dùng AM-GM
ta có $\frac{x}{x^4+y+z}\doteq \frac{1}{x^3+\frac{1}{x}+\frac{1}{x}+\frac{1}{x}-1}\leq \frac{1}{3}$
tương tự ta sẽ có dpcm
ĐTXR khi $x=y=z=1$

#26
no matter what

no matter what

    Why not me

  • Thành viên
  • 397 Bài viết

xin lỗi mình nhầm dấu mình đã sửa lại rồi đó
C2 dùng AM-GM
ta có $\frac{x}{x^4+y+z}\doteq \frac{1}{x^3+\frac{1}{x}+\frac{1}{x}+\frac{1}{x}-1}\leq \frac{1}{3}$
tương tự ta sẽ có dpcm
ĐTXR khi $x=y=z=1$

mình sẽ xem xét cách giải này,hình như có 1 lớp BĐT cũng có chung cách giải trên, :icon6: (dĩ nhiên đâu là cách giải hay)

Mình lại c/m được $\sum \frac{x}{x^4+y+z}\leq 1\ (*)$ Hình đã gửi
---
Vời giả thiết trên thì: $\dfrac{x}{x^4+y+z}=\dfrac{x}{x^4+3-x}$
Ta sẽ c/m: $\dfrac{x}{x^4+3-x}\le \dfrac{1}{3}\Leftrightarrow x^4-4x+3\ge 0\Leftrightarrow (x-1)^2(x^2+2x+3)\ge 0$ (Luôn đúng)
C/m tt rồi cộng lại ta suy ra $(*)$ đúng.
Dấu bằng xảy ra khi $x=y=z=1\ \square$
---
Thôi đi ngủ đây, lần sau bạn nhớ post bài nào liên quan tới topic nhé Hình đã gửi

Qủa này là hệ số bất định phải không minhtuyb hầy? :wacko:

#27
minhtuyb

minhtuyb

    Giả ngu chuyên nghiệp

  • Thành viên
  • 470 Bài viết

a,Hãy suy xét một bài toán cơ bản
Ta xét BĐT sau:Với mọi a,b,c dương có tổng bằng 3
$\frac{1}{a^2(1+a)}+\frac{1}{b^2(1+b)}+\frac{1}{c^2(1+c)}\geq \frac{3}{4abc}$(Olimpic 30-4)
Giải:Ta hoàn toàn có thể giải quyết bài toán trên bằng đánh giá thuận C-S,tuy nhiên,ta sẽ xem xét 1 lời giải sau
BĐT
$\Leftrightarrow \frac{abc}{a^2(1+a)}+\frac{abc}{b^2(1+b)}+\frac{abc}{c^2(1+c)}\geq \frac{3}{4}$
$\Leftrightarrow \frac{bc(b+c)}{a\left [ (b+c)^2+2a(b+c) \right ]} +\frac{ca(c+a)}{b\left [ (c+a)^2+2b(c+a) \right ]}+\frac{ab(a+b)}{c\left [ (a+b)^2+2c(a+b) \right ]}\geq \frac{3}{4}$
$\Leftrightarrow \frac{bc(b+c)}{2a\left [ (b^2+c^2+a(b+c) \right ]} +\frac{ca(c+a)}{2b\left [ a^2+c^2+b(c+a) \right ]}+\frac{ab(a+b)}{2c\left [ a^2+b^2+c(a+b) \right ]}\geq \frac{3}{4}$
$\Leftrightarrow \frac{(b+c)}{2a(\frac{a+b}{c}+\frac{a+c}{b})} +\frac{(c+a)}{2b(\frac{b+c}{a}+\frac{a+b}{c})}+\frac{(a+b)}{2c(\frac{b+c}{a}+\frac{c+a}{b})}\geq \frac{3}{4}$

Bằng việc đặt $\frac{a+b}{c}=x,\frac{b+c}{a}=y,\frac{c+a}{b}=z$,ta dễ dàng đưa BĐT trên về nesbitt 3 biến

Yêu cầu cậu đặt lại phép suy luận toán học ở bài này! Có chỗ là dấu $\Leftarrow$ chứ không phải $\Leftrightarrow$.

c,Làm mạnh BĐT

Tư tưởng giải 2 bài này giống như bài trên nên các bạn luyện tập thêm
"Cũng chẳng có gì khó cả,cái này bình thường thôi"-tôi tin là bạn đang nghĩ vậy,tuy nhiên,bạn hãy xem thêm 2 VD sau để chắc chắn cái bạn nói là đúng nhé
Từ BĐT AM-GM 3 số,ta có thể làm mạnh lên thành
$\frac{a+b+c}{3}\geq \sqrt[3]{abc}+\frac{(a-b)^2+(b-c)^2+(c-a)^2}{12(a+b+c)}$
Cũng cần nói thêm là BĐT này cần tới S.O.S để giải (quá ảo) :lol:

Chưa thử c/m BĐT này bằng $S.O.S$. Nhưng mình cũng xin giới thiệu một BĐT khác cũng khá giống cái này, nhưng được làm mạnh trực tiếp từ $AM-GM$:
$\dfrac{a+b+c}{3}\geq \sqrt[3]{abc}+\dfrac{(\sqrt{a}-\sqrt{b})^2+(\sqrt{b}-\sqrt{c})^2+(\sqrt{c}-\sqrt{a})^2}{3}$

Qủa này là hệ số bất định phải không minhtuyb hầy? :wacko:

Cũng không hẳn. Mình dùng pp tiếp tuyến thôi cho nhanh (bấm máy ra luôn đạo hàm tại $x=1$ là $0$ ) :D
Phấn đấu vì tương lai con em chúng ta!

#28
CelEstE

CelEstE

    Trung sĩ

  • Thành viên
  • 126 Bài viết
Em đóng góp một bất đẳng thức nhỏ.
Chứng minh rằng với mọi số thực dương a,b,c thỏa mãn điều kiện a+b+c=1 thì:
$\sqrt {\frac{{ab}}{{c + ab}}} + \sqrt {\frac{{bc}}{{a + bc}}} + \sqrt {\frac{{ab}}{{c + ab}}} \le \frac{3}{2}$

Freedom Is a State of Mind


#29
Waiting for you

Waiting for you

    Binh nhất

  • Thành viên
  • 37 Bài viết

Em đóng góp một bất đẳng thức nhỏ.
Chứng minh rằng với mọi số thực dương a,b,c thỏa mãn điều kiện a+b+c=1 thì:
$\sqrt {\frac{{ab}}{{c + ab}}} + \sqrt {\frac{{bc}}{{a + bc}}} + \sqrt {\frac{{ab}}{{c + ab}}} \le \frac{3}{2}$

Với điều kiện a+b+c=1,theo AM-GM,ta có
$\sqrt{\frac{ab}{c+ab}}=\sqrt{\frac{ab}{c(a+b+c)+ab}}= \sqrt{\frac{a.b}{(c+a).(c+b)}}\leq \frac{1}{2}(\frac{a}{c+a}+\frac{b}{c+b})$
Xây dựng các BĐT tương tự ta có dpcm

#30
19kvh97

19kvh97

    Sĩ quan

  • Thành viên
  • 423 Bài viết
bài nữa nhé
Bài 7: Cho $a,b,c>0$. CMR
$\frac{a^4}{b^2(b+c)}+\frac{b^4}{c^2(c+a)}+\frac{c^4}{a^(a+b)}\geq \frac{a+b+c}{2}$ :icon6:

#31
minhtuyb

minhtuyb

    Giả ngu chuyên nghiệp

  • Thành viên
  • 470 Bài viết

bài nữa nhé
Bài 7: Cho $a,b,c>0$. CMR
$\frac{a^4}{b^2(b+c)}+\frac{b^4}{c^2(c+a)}+\frac{c^4}{a^(a+b)}\geq \frac{a+b+c}{2}$ :icon6:


Đề nghị các bạn giải quyết hết bài tồn động trước khi quăng bài mới :ph34r:
---
$$\dfrac{a^4}{a^2(b+c)}+\dfrac{a}{2}+\dfrac{a}{2}+\dfrac{b+c}{4}\ge 2a$$
Tương tự rồi cộng lại

Do phần này khá rộng nên mình xin trình bày bài tập riêng (hơi nhiều 1 tí) :icon6:
1,Chứng minh với mọi a,b,c $0\leq a,b,c\leq \frac{1}{2}$ thỏa mãn $a+b+c=1$ thì
$\frac{1}{a(2b+2c-1)}+\frac{1}{b(2c+2a-1)}+\frac{1}{c(2a+2b-1)}\geq 27$



Bài này mà $AM-GM$ được thì cũng hơi xoắn :o
---
$$\frac{1}{a(2b+2c-1)}=\dfrac{1}{a(1-2a)}$$
Sau đó ta đi c/m BĐT phụ:
$$\dfrac{1}{a(1-2a)}\ge 27a\\ \Leftrightarrow (3a-1)^2(6a+1)\ge 0$$

2,Chứng minh với mọi a,b,c $\geq \frac{1}{2}$,ta có
$\frac{a^2}{\sqrt{5-2(b+c)}}+\frac{b^2}{\sqrt{5-2(c+a)}}+\frac{c^2}{\sqrt{5-2(a+b)}}\geq 3$


$$\dfrac{a^2}{\sqrt{5-2(b+c)}}\ge \dfrac{(\dfrac{1}{2})^2}{\sqrt{5-2(\dfrac{1}{2}+\dfrac{1}{2})}}=\dfrac{\sqrt{3}}{12}$$
Tương tự rồi cộng lại ta thấy $VT\ge \dfrac{\sqrt{3}}{4}$

3,Chứng minh với mọi a,b,c có $0< a,b,c\leq \frac{1}{3},a^3+b^3+c^3=\frac{3}{64}$ thì
$\frac{1}{1-3a}+\frac{1}{1-3b}+\frac{1}{1-3c}\geq 12$


Tư tưởng tương tự bài $2$, ta xây dựng BĐT phụ:
$$\dfrac{1}{1-3a}\ge 256a^3\\ \Leftrightarrow (4a-1)^2(48a^2+8a+1)\ge 0\ (True)$$
Phấn đấu vì tương lai con em chúng ta!

#32
Waiting for you

Waiting for you

    Binh nhất

  • Thành viên
  • 37 Bài viết

Đề nghị các bạn giải quyết hết bài tồn động trước khi quăng bài mới :ph34r:
---
$$\dfrac{a^4}{a^2(b+c)}+\dfrac{a}{2}+\dfrac{a}{2}+\dfrac{b+c}{4}\ge 2a$$
Tương tự rồi cộng lại




Bài này mà $AM-GM$ được thì cũng hơi xoắn :o
---
$$\frac{1}{a(2b+2c-1)}=\dfrac{1}{a(1-2a)}$$
Sau đó ta đi c/m BĐT phụ:
$$\dfrac{1}{a(1-2a)}\ge 27a\\ \Leftrightarrow (3a-1)^2(6a+1)\ge 0$$

Phải chăng còn 1 cách bằng AM-GM kiểu này :wub:
Áp dụng trực tiếp AM-GM cho 3 số ta có
$VT\geq \frac{3}{\sqrt[3]{abc(2b+2c-1)(2c+2a-1)(2a+2b-1)}}$
Vậy ta chỉ cần chứng minh
$9\sqrt[3]{abc(2b+2c-1)(2c+2a-1)(2a+2b-1)}\leq 1$
tuy nhiên có thể thấy BĐT này đúng theo AM-GM
$\sqrt[3]{abc}\leq \frac{a+b+c}{3}=\frac{1}{3}$
$\sqrt[3]{(2b+2c-1)(2c+2a-1)(2a+2b-1)}\leq \frac{4(a+b+c)-3}{3}=\frac{1}{3}$

#33
CelEstE

CelEstE

    Trung sĩ

  • Thành viên
  • 126 Bài viết

Với điều kiện a+b+c=1,theo AM-GM,ta có
$\sqrt{\frac{ab}{c+ab}}=\sqrt{\frac{ab}{c(a+b+c)+ab}}= \sqrt{\frac{a.b}{(c+a).(c+b)}}\leq \frac{1}{2}(\frac{a}{c+a}+\frac{b}{c+b})$
Xây dựng các BĐT tương tự ta có dpcm

Cũng có thể phân tích các mẫu ví dụ như $c+ab=1-a-b+ab=(1-a)(1-b)$

Freedom Is a State of Mind


#34
CelEstE

CelEstE

    Trung sĩ

  • Thành viên
  • 126 Bài viết
Thật không vậy? :)) làm ra nó cũng tương tự như cách của bạn.
\[\begin{array}{l}
\sqrt {\frac{{ab}}{{c + ab}}} = \sqrt {\frac{{ab}}{{(1 - a)(1 - b)}}} \le \frac{1}{2}(\frac{a}{{1 - b}} + \frac{b}{{1 - a}})\\
\sqrt {\frac{{bc}}{{a + bc}} \le } \frac{1}{2}(\frac{b}{{1 - c}} + \frac{c}{{1 - b}})\\
\sqrt {\frac{{ca}}{{b + ca}} \le } \frac{1}{2}(\frac{c}{{1 - a}} + \frac{a}{{1 - c}})
\end{array}\]
Cộng lại bất đẳng thức được chứng minh.

Bài viết đã được chỉnh sửa nội dung bởi CelEstE: 27-11-2012 - 17:45

Freedom Is a State of Mind


#35
Oral1020

Oral1020

    Thịnh To Tướng

  • Thành viên
  • 1225 Bài viết
Mình góp bài nhé
Cho $a,b,c>0$ và $a+b+c=3$.Chứng minh rằng
$a^2+b^2+c^2+\dfrac{ab+bc+ac}{a^2b+b^2c+c^2a} \ge 4$

"If I feel unhappy,I do mathematics to become happy.


If I feel happy,I do mathematics to keep happy."

Alfréd Rényi

Hình đã gửi


#36
Waiting for you

Waiting for you

    Binh nhất

  • Thành viên
  • 37 Bài viết

Mình góp bài nhé
Cho $a,b,c>0$ và $a+b+c=3$.Chứng minh rằng
$a^2+b^2+c^2+\dfrac{ab+bc+ac}{a^2b+b^2c+c^2a} \ge 4$

minhtuyb said:Đề nghị các bạn giải quyết hết bài tồn động trước khi quăng bài mới :ph34r:
no matter what said:tạm thời mong mọi người tham gia XỬ LÝ các bài trong topic trước,khi nào phần kiến thức post hết,mọi người hãy thoải mái post bài ngoài.Mình thật sự xin lỗi vì sự bất tiện này và cũng mong mọi người cùng giúp để topic phát triển -Xin chân thành cảm ơn
mình nghĩ ta cứ làm như vậy đã nhỉ??? :icon6:

#37
Waiting for you

Waiting for you

    Binh nhất

  • Thành viên
  • 37 Bài viết
Chủ topic sao không thấy post bài nữa nhỉ? :wacko:

#38
hocgiotto13031998

hocgiotto13031998

    Lính mới

  • Thành viên
  • 9 Bài viết

em xem lại đề bài 1 xem hình như thừa thì phải
đề đúng là $\frac{1}{a^2+2bc}+\frac{1}{b^2+2ca}+\frac{1}{c^2+2ab}\geq 9$ phải không em?

cau a, ap dung cosi cho 2 so la dc
cau b, ap dung bu nhi a cot xki la xong

#39
pcfamily

pcfamily

    Thượng sĩ

  • Thành viên
  • 212 Bài viết

Sau đó ta đi c/m BĐT phụ:
$$\dfrac{1}{a(1-2a)}\ge 27a\\ \Leftrightarrow (3a-1)^2(6a+1)\ge 0$$


Cái nầy chả là AM-GM là gì :-P

$\frac{1}{a(1-2a)}=\frac{a}{a.a(1-2a)}\geq \frac{a}{\frac{1}{27}}$

Bài viết đã được chỉnh sửa nội dung bởi pcfamily: 28-11-2012 - 21:21


#40
Sagittarius912

Sagittarius912

    Trung úy

  • Thành viên
  • 776 Bài viết
Mình xin đóng góp vài bài:
1) Cho $a,b,c>0$ chứng minh rằng
$\frac{ab}{c^{2}}+\frac{ca}{b^{2}}+\frac{bc}{a^{2}}\geq \frac{1}{2}(\frac{a+b}{c}+\frac{c+a}{b}+\frac{b+c}{a})$
2)Cho $a,b,c>0$ chứng minh rằng
$\frac{b+c}{a^{2}}+\frac{c+a}{b^{2}}+\frac{a+b}{c^{2}}\geq 2(\frac{1}{a}+\frac{1}{b}+\frac{1}{c})$
3)Cho $a,b,c>0$ thoả mãn $a+b+c=3$chứng minh rằng
$abc(a^{2}+b^{2}+c^{2})\leq 3$
4)Cho $a,b,c,d>0$ chứng minh rằng

$\frac{a-b}{b-c}+\frac{b-c}{c+d}+\frac{c-d}{d+a}+\frac{d-a}{a+b}\geq 0$

P/s: lời giải post sau ^^




0 người đang xem chủ đề

0 thành viên, 0 khách, 0 thành viên ẩn danh